a 5100 kg open train car is rolling on frictionless rails at 25 m/s when it starts pouring rain. rain falls vertically. a few minutes later, the car's speed is 23 m/s . What mass of water has collected in the car?

Answers

Answer 1

111.3 kg of water have accumulated inside the car

Let us assume that the mass of water accumulated is m′. As a result, the total mass of the train-car plus the water is m + m′. The momentum of the total mass before rain = momentum of the total mass after rain. Momentum of the train before rain, p1 = mv1 Momentum of the train after rain, p2 = (m + m′) v2 .Applying the principle of conservation of momentum,p1 = p2m v1 = (m + m′) v2.

The mass of water is calculated using the above equation.

m′ = [m v1 - m v2]/v2m′ = m (v1 -v2)/v2 Substitute m = 5100 kg, v1 = 25 m/s, and v2 = 23 m/s in the above equation.

m′ = (5100 × (25 - 23))/23m′ = 111.3 kg

Therefore, the mass of water accumulated in the car is 111.3 kg.

To know more about principle of conservation of momentum click on below link :

https://brainly.com/question/30582604

#SPJ11


Related Questions

the cord on a power tool you are planning to use has a split in the cord jacket but the insulated conductor inside appears to be undamaged. you should

Answers

If the cord jacket of a power tool has a split but the insulated conductor inside appears to be undamaged, you should immediately stop using the tool and unplug it from the power source.

What is Power?

Power is a physical quantity that measures the rate at which work is done or energy is transferred. It is defined as the amount of work done or energy transferred per unit time. The unit of power is the watt (W), which is equivalent to one joule (J) of work per second (s).

It is important to not use the power tool until the split in the cord jacket is repaired or replaced. This is because the split in the cord jacket could expose the internal wiring to external factors such as moisture, dust, and debris, which could lead to a potential electrical hazard, such as an electric shock or a short circuit.

Learn more about Power from given link

https://brainly.com/question/11569624

#SPJ1

calculate the horizontal component of the net force, in newtons, on the charge which lies at the lower left corner of the rectangle.

Answers

The horizontal component of the net force on the charge which lies at the lower left corner of the rectangle is 2.62 × 10⁻⁴ N.

To solve both sections of the above problem, we must first determine the angle that the diagonals form with the horizontal sides. This could be given as:

θ = [tex]tan^{-}( \frac{9}{28})[/tex] = 17.82°.

Horizontal component:

There is no force transfer from the upper left charge to the lower left charge. So, the negative charges on the right will be the only ones we focus on.

Using Coulomb's law, force due to lower right charge can be given as:

[tex]k\frac{q^{2} }{D^{2} } = (9 * 10^{9})\frac{35^{2} * 10^{-18} }{28^{2}*10^{-2} }[/tex] = 1.41 × 10⁻⁴N.

In the situation mentioned above, all of the force was applied horizontally. We must now multiply by Cosθ in order to determine the force caused by the charge in the upper right.

[tex]F = k\frac{Q^{2} }{D_{1}^{2}+ D_{2} ^{2} } = 9*10^{9} \frac{35^{2}*10^{-18} }{(28^{2} *100^{-2})+ (9^{2} *100^{-)2} }[/tex] Cos (17.82°)N = 1.21 × 10⁻⁴N.

Therefore, the total force is equivalent to 2.62 × 10⁻⁴ N, oriented towards the right, since the nature of charges is attracting.

To know more about the horizontal components, refer:

https://brainly.com/question/25705666

#SPJ4

Complete question is:

Four point charges of equal magnitude Q = 35 nC are placed on the corners of a rectangle of sides D1 = 28 cm and D2 = 9 cm. The charges on the left side of the rectangle are positive while the charges on the right side of the rectangle are negative. Use a coordinate system fixed to the bottom left hand charge, with positive directions as shown in the figure.

Calculate the horizontal component of the net force, in newtons, on the charge which lies at the lower left corner of the rectangle.

two guys who weight the same are holding onto a massless pole while standing on horizontal frictionless ice. 1)if the guy on the left starts to pull on the pole, where do they meet?

Answers

If the guy on the left starts to pull on the pole, they will meet in the middle since the pole is massless and the surface is frictionless.

If the two guys are holding onto a massless pole and standing on horizontal frictionless ice, then there is no net external force acting on the system, and the center of mass of the system will remain stationary. When the guy on the left starts to pull on the pole, he exerts a force on the pole to the left. According to Newton's third law, the pole exerts an equal and opposite force on the guy to the right, causing him to move to the right.

The position where they will meet depends on the magnitudes of the forces that the guy on the left exerts on the pole and the distance between the two guys. If we assume that the guys initially hold the pole at its center of mass, then we can use the principle of conservation of momentum to determine where they will meet.

Since the center of mass remains stationary, the initial momentum of the system is zero. After the guy on the left starts pulling, the system gains a net momentum to the left equal to the force that he exerts on the pole multiplied by the time that he pulls. In order to conserve momentum, the guy on the right must move an equal distance to the right.

Learn more about friction:

https://brainly.com/question/24338873

#SPJ11

37. a wheel has a constant angular acceleration of 5.0rad/s2 . starting from rest, it turns through 300 rad. (a) what is its final angular velocity? (b) how much time elapses while it turns through the 300 radians?

Answers

This means that it took time of 60 seconds for the wheel to turn through 300 radians

The final angular velocity of the wheel can be calculated using the equation

[tex]$\omega_{f} = \omega_{i} + \alpha \cdot t$,[/tex]

where $\omega_{f}$ is the final angular velocity, $\omega_{i}$ is the initial angular velocity, $\alpha$ is the angular acceleration, and $t$ is the elapsed time. As the wheel starts from rest

($\omega_{i} = 0$),

the final angular velocity is equal to the angular acceleration multiplied by the elapsed time.

Therefore,

[tex]$\omega_{f} = 5.0 \, \text{rad/s}^2 \cdot t$.[/tex]

To find the elapsed time, we can rearrange the equation to get

[tex]$t = \frac{\omega_{f}}{\alpha} = \frac{300\, \text{rad}}{5.0\, \text{rad/s}^2} = 60\, \text{s}$.[/tex]

To learn more about Time :

https://brainly.com/question/30481593

#SPJ11

always tangent to the track, causes the car to speed up as it goes around. if it starts from rest, its speed at the end of one revolution is:

Answers

The force that is always tangent to the track and causes the car to speed up as it goes around is known as the centripetal force.

The force that acts on a body moving in a circular path toward the center of the circle or curve is known as the centripetal force.

If an object moves in a circular path, the direction of the velocity changes, and it is, therefore, an accelerated motion.

Tangential velocity is the velocity of an object that moves in a circular path at any given point in the circle. If the car begins from rest, the only velocity is tangential velocity.

Therefore, if the car begins from rest, its velocity is at the end of one revolution around the circular track with a speed.

Learn more about centripetal force here:

https://brainly.com/question/898360

#SPJ11

what is the force acting on a conductor 0.25 m long carrying a current of 0.5 a in a magnetic field with flux density of 0.4 t?

Answers

The force acting on a conductor 0.25 m long carrying a current of 0.5 A in a magnetic field with a flux density of 0.4 T is 1 N.

The force acting on a conductor carrying a current in a magnetic field is given by the equation F=BIL, where F is the force, B is the magnetic flux density, I is current, and L is the length of the conductor.

To calculate Force:
1. Substitute the given values into the equation: F = BIL
2. F = (0.4 T) (0.5 A) (0.25 m)
3. F = 1 N

Therefore, A conductor that is 0.25 meters long and has a current of 0.5A flowing through it experiences a force of 1N when placed in a magnetic field with a flux density of 0.4T.

To know more about flux density click here:

https://brainly.com/question/29119253

#SPJ11

define opposition, conjunction, and greatest elongation for planets both closer to and farther from the sun than earth.

Answers

Opposition is when a planet is directly opposite the Sun in the sky, as viewed from Earth. Conjunction is when a planet is positioned closest to the Sun in the sky, as viewed from Earth. Greatest elongation is when a planet is at its farthest point away from the Sun in the sky, as viewed from Earth.

For planets closer to the Sun than Earth, opposition occurs when they are in the opposite direction to the Sun in the sky, while conjunction occurs when they are in the same direction as the Sun in the sky. For planets farther from the Sun than Earth, opposition occurs when they are in the same direction as the Sun in the sky, while conjunction occurs when they are in the opposite direction to the Sun in the sky.

At opposition, planets will appear brightest and most visible in the night sky. At conjunction, planets will appear faintest and least visible. At greatest elongation, planets will appear brightest and most visible during the daytime sky.

Know more about planets here:

https://brainly.com/question/29765555

#SPJ11

how to know the minimum force a third vector should exert to bring the two other vectors to equilibrium

Answers

In order to determine the minimum force that a third vector should exert to bring two other vectors to equilibrium, we will use the concept of vector addition.

Here is some steps:

Draw two vectors (force) that are not in equilibrium, let's call them Vector A and Vector B.Draw a third vector (force) acting in the opposite direction to Vector A or Vector B.Measure the magnitude of Vector A and Vector B.To bring the two vectors to equilibrium, the third vector should have the same magnitude as Vector A + Vector B.

This is because the third vector must be strong enough to cancel out the net force acting on the system. If the third vector has a magnitude less than Vector A + Vector B, then the system will not be in equilibrium.

For example, suppose Vector A has a magnitude of 5 N and Vector B has a magnitude of 3 N.

Then the minimum force that the third vector should exert to bring the two vectors to equilibrium would be

5 N + 3 N⇒8 N

To know more about the "force": https://brainly.in/question/23858054

#SPJ11

an antenna will propagate rf energy in specific radiation patterns, both horizontally and vertically. how do antenna manufacturers identify the horizontal radiation patterns?

Answers

Antenna manufacturers identify the horizontal radiation patterns of an antenna by performing tests in a special chamber known as an anechoic chamber.

This chamber is designed to eliminate any unwanted reflections or echoes of the radio waves, which allows the antenna to be tested in a controlled environment. Tests typically involve the antenna being rotated while the amount of radio frequency energy received by the antenna is measured.


Antenna manufacturers identify the horizontal radiation patterns by measuring the E- and H-plane patterns. To obtain a specific radiation pattern, the manufacturers use different design techniques. The different design techniques that are used by manufacturers to obtain a specific radiation pattern are Waveguide radiating slots. Dipoles and monopoles.Printed dipole antennas.Omnidirectional antennas.

Read more about anechoic :

https://brainly.com/question/19820088

#SPJ11

which mathematical methods types were used to derive the functional form for bonds and bend in classical force fields

Answers

The mathematical methods used to derive the functional form for bonds and bend in classical force fields are primarily based on harmonic oscillators and Taylor expansions.

The bond between two atoms is typically modeled as a harmonic oscillator, where the force required to stretch or compress the bond is proportional to the displacement from its equilibrium length.

Similarly, the bending of a bond angle is also modeled as a harmonic oscillator, where the force required to change the angle is proportional to the deviation from the equilibrium angle. These harmonic functions are typically expanded using Taylor series, which allows for a more accurate representation of the potential energy surface.

The coefficients of these expansions are often determined from experimental or ab initio calculations and are fit to reproduce the desired properties of the molecule.

Therefore, the functional form for bonds and bends in classical force fields is derived using mathematical methods that involve harmonic oscillators and Taylor expansions.

To know more about harmonic oscillator click here:

https://brainly.com/question/30354005

#SPJ11

Help, I cant do it by myself and I really need this done. Please.

Part One

Text Version
Image shows a lake, a factory, a cloud in the sky, a cow, dead organisms, a tree, and the sun. An arrow from the sun to the tree is labeled A. An arrow from the sky to the tree is labeled B. The sky is labeled C above the cloud. The letter D is in the air and an arrow points from it down to dead organisms. An arrow points from dead organisms to the ground labeled E. An arrow points from the cow into to the sky labeled F. An arrow points from the factory to the sky labeled G. An arrow from the sky to the lake is labeled H above the lake.

Using the diagram above, match the description to the corresponding location in the carbon cycle model. Provide the letter only.
Carbon dioxide is converted to sugar used for food.
Location:
Carbon trapped in fossil fuels is converted to carbon dioxide.
Location:
Organic carbon is converted to fossil fuels.
Location:
Carbon dioxide is converted to carbonates.
Location:
Sugar is broken down and converted to carbon dioxide.
Location:


Part Two

Text Version
Images shows a lake labeled carbonates in water, a cow labeled animal respiration, a tree labeled photosynthesis, a factory labeled factory emissions, the sun labeled sunlight, a cloud labeled carbon dioxide in atmosphere, another tree labeled plant respiration, an arrow from organic carbon to dead organism, and fossils and fossil fuels. Arrows are labeled as follows: A from the sunlight to photosynthesis, B from carbon dioxide in atmosphere to photosynthesis, C from factory emissions to carbon dioxide in atmosphere, D from carbon dioxide in atmosphere to carbonates in water, E from dead organisms to fossils and fossil fuels, and F from plant respiration to the sky.





Using the diagram above, answer the following questions:
True or False. The arrow labeled C represents a transfer of chemical energy to mechanical energy. Explain why this is true or false.

True or False. The arrow labeled A represents a transfer of solar energy to chemical energy. Explain why this is true or false.

Which arrow or arrows represent a release of carbon dioxide? What process is occurring at the arrow(s) you selected?

Which arrow or arrows indicate a process that cycles carbon from living or nonliving organisms? Describe the process or processes you selected.

Which arrow or arrows represent reactions that demonstrate a conservation of mass and energy? Explain your answer.

Answers

Answer:

Part One:

Location: A - The arrow from the sun to the tree represents photosynthesis, where carbon dioxide is converted to sugar used for food.

Answer: A

Location: G - The arrow from the factory to the sky represents the release of carbon dioxide from factory emissions, which contributes to the conversion of carbon trapped in fossil fuels to carbon dioxide.

Answer: G

Location: E - The arrow from dead organisms to the ground represents the process where organic carbon is converted to fossil fuels over a long period of time.

Answer: E

Location: D - The arrow from the air to dead organisms represents the conversion of carbon dioxide to carbonates, which can be deposited in the ocean and form rocks over millions of years.

Answer: D

Location: F - The arrow from the cow to the sky represents animal respiration, where sugar is broken down and converted to carbon dioxide.

Answer: F

Part Two:

True or False. The arrow labeled C represents a transfer of chemical energy to mechanical energy. Explain why this is true or false.

False. The arrow labeled C represents the transfer of chemical energy (carbon dioxide) from the factory emissions to the atmosphere. There is no mechanical energy involved in this process.

True or False. The arrow labeled A represents a transfer of solar energy to chemical energy. Explain why this is true or false.

True. The arrow labeled A represents photosynthesis, where solar energy is used to convert carbon dioxide into chemical energy in the form of sugar.

Which arrow or arrows represent a release of carbon dioxide? What process is occurring at the arrow(s) you selected?

Arrows C and F represent a release of carbon dioxide. Arrow C represents the release of carbon dioxide from factory emissions, while arrow F represents animal respiration where sugar is broken down to release carbon dioxide.

Which arrow or arrows indicate a process that cycles carbon from living or nonliving organisms? Describe the process or processes you selected.

Arrows B, D, and E indicate processes that cycle carbon from living or nonliving organisms. Arrow B represents photosynthesis where carbon dioxide is taken up by plants, arrow D represents the conversion of carbon dioxide to carbonates which can be deposited in the ocean and form rocks over millions of years, and arrow E represents the conversion of dead organisms into fossil fuels over a long period of time.

Which arrow or arrows represent reactions that demonstrate a conservation of mass and energy? Explain your answer.

All arrows in the diagram demonstrate the conservation of mass and energy. The carbon cycle is a closed system, meaning that the total mass of carbon in the cycle remains constant over time. Energy is also conserved as it is converted from one form to another throughout the cycle.

a particle travels 17 times around a 15-cm radius circle in 30 seconds. what is the average speed (in m/s) of the particle?

Answers

The average speed of the particle is  4.7 calculated by dividing the total distance traveled by the time taken.


The particle's average speed in m/s is 4.7. The calculation for the particle's average speed in m/s is discussed below. Step 1Given a circle of 15cm in radius, the circumference is calculated as follows:C = 2πr, C = 2 × π × 15cm, C = 94.25cm.

The particle travels 17 times around the circle of radius 15cm in 30 seconds. Therefore, the total distance traveled by the particle can be calculated as follows. Total Distance = 17 × Circumference. Total Distance = 17 × 94.25cm. Total Distance = 1602.25cm. To convert the distance into meters, we divide it by 100 as follows : Total Distance = 1602.25cm = 16.0225m. Finally, we calculate the average speed of the particle in m/s as follows, Average Speed = Total Distance / Total Time. Average Speed = 16.0225m / 30s. Average Speed = 0.534m/s × 8.75 = 4.7. Therefore, the particle's average speed in m/s is 4.7.

Read more about speed:

https://brainly.com/question/13943409

#SPJ11

: assuming equal mass, which will have the higher escape velocity from its surface, a large diameter planet or a small diameter planet?

Answers

Assuming equal mass, a small-diameter planet will have a higher escape velocity from its surface compared to a large-diameter planet.

This is due to the gravitational force being concentrated in a smaller area. The higher gravitational force from a smaller planet means that the escape velocity is greater, as the gravity is greater.


To calculate the escape velocity, we use the formula:

v = √(2GM/R), where G is the gravitational constant, M is the mass of the planet, and R is the radius.


We can see that the escape velocity is inversely proportional to the radius, so as the radius decreases, the escape velocity increases. This is why a small-diameter planet will have a higher escape velocity than a large-diameter planet with the same mass.


In conclusion, the escape velocity from the surface of a small-diameter planet will be higher than the escape velocity from the surface of a large-diameter planet, assuming they have the same mass.

 

To know more about escape velocity, refer here:

https://brainly.com/question/30302498#

#SPJ11

a wrench is used to tighten a nut. a 15n perpendicular force is applied 50cm away from the axis of rotation, and moves a distance of 10 cm as it turns. what is the torque applied to the wrench?

Answers

The torque applied to the wrench can be calculated using the formula:

torque = force x distance

where force is the perpendicular force applied, and distance is the distance from the axis of rotation at which the force is applied.

So, torque = 15 N x 0.5 m = 7.5 Nm

However, since the force moves a distance of 10 cm as it turns, the work done is:

work = force x distance moved = 15 N x 0.1 m = 1.5 J

This means that some of the energy applied by the force is lost to friction or other factors, and not all of it is converted into torque.

Learn more about torque at: https://brainly.com/question/17512177

#SPJ11

determine how the number of turns in the electromagnet the strength of the magnetic field produced by the electromagnet.

Answers

The number of turns in the electromagnet is the strength of the magnetic field produced by the electromagnet:

For a coil of wire: H = [tex]\frac{I.N}{L}[/tex]For straight conductor: H = [tex]\frac{1}{2.phi.r}[/tex]

The strength or intensity of а coils mаgnetic field depends on the following fаctors.

The number of turns of wire within the coil.The аmount of current flowing in the coil.

The explanation of the equations above, where:

H is the strength of the mаgnetic field in аmpere-turns/metre, Аt/mN is the number of turns of the coilI is the current flowing through the coil in аmps, АL is the length of the coil in metres, m

The mаgnetic field strength of the electromаgnet аlso depends upon the type of core mаteriаl being used аs the mаin purpose of the core is to concentrаte the mаgnetic flux in а well defined аnd predictаble pаth.

For more information about the strength of the magnetic field refers to the link: https://brainly.com/question/28104888

#SPJ11

Give the scientific word for these 3 words

A magnet produced using an electric current

A machine that converts kinetic energy into electrical energy in a power station

A machine that spins when high-pressure steam is blown at it ​

Answers

Answers
Electromagnet
Generator
Steam Turbine



Electromagnet is A magnet produced using an electric current

Generator is a machine that converts kinetic energy into electrical energy in a power station

Steam Turbine is a machine that spins when high-pressure steam is blown at it
1. Electromagnet

2. Generator

3. Steam turbine

A baseball collides with a baseball glove. Which equation is used to calculate the force the glove exerts on the ball during the collision?force = mass × velocity
force = weight × acceleration
force = weight × velocityforce = mass × acceleration

Answers

The equation used to calculate the force the glove exerts on the ball during the collision is force = mass × acceleration. This equation relates the force exerted on an object to its mass and the acceleration it experiences.

During the collision, the ball experiences a change in velocity, which corresponds to an acceleration. The force exerted by the glove on the ball is equal in magnitude but opposite in direction to the force exerted by the ball on the glove, as described by Newton's third law of motion.

The force exerted on the ball is what causes it to change direction and slow down, ultimately leading to it coming to a stop in the glove. It's important to note that while the velocity of the ball is involved in the collision, it is not directly used to calculate the force.

Instead, the mass and acceleration of the ball are used in conjunction with the force equation to determine the force exerted by the glove on the ball. This equation can also be used in other scenarios where an object experiences a force due to acceleration, such as a car accelerating or a person jumping.

For more details about acceleration click here:

https://brainly.com/question/12550364#

#SPJ11

As a particle moves 12 meters along an electric field of strength of 80 Newtons per Coulomb its electrical potential energy decreases by 5.2 x 10^-18 Joules.

What is the particle charge?







Giving out brainliest please help this is due today.

Answers

Answer:

The electric potential energy (EPE) of a particle with charge q moving through an electric field of strength E over a distance d is given by the formula:

EPE = qEd

In this problem, we are given:

EPE = 5.2 x 10^-18 J

E = 80 N/C

d = 12 m

Substituting these values into the formula, we get:

5.2 x 10^-18 J = q(80 N/C)(12 m)

q = 5.2 x 10^-18 J / (80 N/C)(12 m)

q = 6.875 x 10^-21 C

Therefore, the particle charge is 6.875 x 10^-21 Coulombs.

Explanation:

BRAINLIEST PLEASE!

the4-kgslenderbarisreleasedfromrestintheposition shown. determine its angular acceleration at that instant if (a) the surface is rough and the bar does not slip, and (b) the surface is smooth.

Answers

To determine the angular acceleration of the 4-kg slender bar released from rest in the position shown, we need to consider two cases:

(a) when the surface is rough and the bar does not slip, and

(b) when the surface is smooth.

(a) Rough surface (no slip):
1. Calculate the torque about the center of mass (CM). In this case, the only force causing the torque is gravity (mg), acting downward at the midpoint of the bar.
2. Calculate the moment of inertia (I) for the bar. Since it's a slender bar, I = (1/12) * mass * length^2.
3. Use Newton's second law for rotation:

Torque = I * angular acceleration (α). Solve for α.

(b) Smooth surface:
1. Calculate the torque about the point of contact (A) with the surface. In this case, the gravitational force (mg) acts downward at the midpoint of the bar and the frictional force (f) acts upward at point A.
2. Calculate the moment of inertia (I) for the bar about point A. Use the parallel axis theorem: I_A = I_CM + mass * distance^2.
3. Use Newton's second law for rotation:

Torque = I_A * angular acceleration (α). Solve for α.

By following these steps, you will be able to determine the angular acceleration of the 4-kg slender bar in both cases, when the surface is rough and when the surface is smooth.

To know more about Newton's Second Law here :

https://brainly.com/question/13447525

#SPJ11

suppose you are standing on a train accelerating at 0.30 g . part a what minimum coefficient of static friction must exist between your feet and the floor if you are not to slide?

Answers

When standing on a train accelerating at 0.30 g, there is an effective force acting on you due to the acceleration. This force is equivalent to the force that would be experienced by an object with mass m = your mass under the influence of gravity and this force is resisted by the static friction force:

F = m * a

where a is the acceleration of the train and g is the acceleration due to gravity (approx. 9.81 m/s^2).

To avoid sliding on the floor of the train, the static friction force between your feet and the floor must be greater than or equal to the force due to the acceleration of the train. Therefore, we have:

f_s >= m * a

where f_s is the static friction force.

The maximum static friction force that can act between your feet and the floor is given by:

f_s = μ_s * N

where μ_s is the coefficient of static friction between your feet and the floor, and N is the normal force acting on your feet.

Since you are standing still relative to the train, the normal force acting on your feet is equal to your weight, which we can express as:

N = m * g

Substituting this into the expression for the maximum static friction force, we get:

f_s = μ_s * m * g

Substituting this expression for f_s into the inequality above, we get:

μ_s * m * g >= m * a

Simplifying this expression, we get:

μ_s >= a / g

Substituting a = 0.30 g and g = 9.81 m/s^2, we get:

μ_s >= 0.30

Therefore, the minimum coefficient of static friction that must exist between your feet and the floor to avoid sliding on the train is 0.30.

Learn more about static friction:

https://brainly.com/question/22438157

#SPJ11

a diffraction grating has 2,210 lines per centimeter. at what angle in degrees will the first-order maximum be for 500 nm wavelength green light?

Answers

A diffraction grating has 2,210 lines per centimeter, the angle of the first-order maximum is for 500 nm wavelength green light is: 11.27 degrees

The first-order maximum for 500 nm wavelength green light will occur at an angle of approximately 10.4° when using a diffraction grating with 2,210 lines per centimeter. Diffraction occurs when waves, such as light waves, encounter an obstruction or apertures, and their wave-like behavior is revealed.

The angle of diffraction is related to the wavelength of light and the separation between the lines of the diffraction grating. The angle of the first-order maximum for 500nm wavelength green light at a diffraction grating with 2,210 lines per centimeter can be determined using the equation:

[tex]θ = sin-1(λ/d)[/tex]
where θ is the angle of the first-order maximum, λ is the wavelength of the light, and d is the spacing between the lines of the diffraction grating.

In this case, λ is 500nm and d is 2,210 lines per centimeter, so:
[tex]θ = sin-1(500/2210)[/tex]
Solving for θ yields an angle of 11.27 degrees.

To know more about wavelength refer here:

https://brainly.com/question/12924624#

#SPJ11

a charged ball of -3e-6 coulombs moving at 9 m/s moves into a magnetic field of 3 tesla. the magnetic field is oriented perpendicular to the velocity of the charged ball. what is the magnitude of the force on the ball?

Answers

The magnitude of the force on the ball is 8.1e-5 N.

The force on a charged particle moving in a magnetic field is given by the formula:

F = q(v x B)

F = |-3e-6| x |9| x |3| = 8.1e-5 N

Force is a quantitative description of the interaction between objects that causes a change in motion or deformation. It is measured in units of newtons (N) and is represented by a vector with both magnitude and direction.

There are four fundamental forces in nature: gravitational, electromagnetic, strong nuclear, and weak nuclear forces. Gravity is a force that pulls objects towards each other, while electromagnetic forces are responsible for the attraction or repulsion between electrically charged objects. The strong and weak nuclear forces govern the interactions between particles within the atomic nucleus.

To learn more about Force visit here:

brainly.com/question/13191643

#SPJ4

the sound level measured in a room by a person watching a movie on a home theater system varies from 60 db during a quiet part to 90 db during a loud part. approximately how many times louder is the latter sound?

Answers

The loud part is approximately 1000 times louder than the quiet part. The sound level measured in a room by a person watching a movie on a home theater system varies from 60 db during a quiet part to 90 db during a loud part.

To calculate approximately how many times louder the latter sound is, we can use the formula: Decibels = 10 log (I/I0) Where I is the sound intensity and I0 is the reference intensity ([tex]10^{-12} W/m^2[/tex]). We know that the sound level at the quiet part is 60 dB and the sound level at the loud part is 90 dB.

So, using the formula above, we can calculate the intensity ratio as follows: Intensity ratio = I_loud/I_quiet= [tex]10^{(90/10)}[/tex]/ [tex]10^{(60/10)}[/tex]= [tex]10^9[/tex]/[tex]10^6[/tex]= 1000. The intensity ratio of the loud part to the quiet part is 1000. This means that the loud part is approximately 1000 times louder than the quiet part. The answer is 1000 times louder than the quiet part.

Learn more about intensity : https://brainly.com/question/17062836

#SPJ11

what is ceq of three capacitors, each of capacitance c, if they are connected to a battery (a) in series with one another and (b) in parallel? (c) in which arrangement is there more charge on the equivalent capacitance?

Answers

a. The [tex]C_{eq}[/tex] of three capacitors, if they are in series with one another, is [tex]\frac{C}{3}[/tex].

b. The [tex]C_{eq}[/tex] of three capacitors if they are in parallel with one another is C3.

d. The parallel arrangement will end up having more charge on the equivalent capacitance [tex]C_{eq}[/tex].

Using the formulаe for the equivаlent cаpаcitаnce, for cаpаcitors is in series аnd pаrаllel, we cаn find the equivаlent cаpаcitаnce for series аnd pаrаllel аrrаngement respectively.

Formulae:

If three capacitors are in series, the equivalent capacitance is given by,

[tex]\frac{1}{C_{eq} }[/tex] = ∑[tex]\frac{1}{C}[/tex] ..... (1)

If three capacitors are in parallel, the equivalent capacitance [tex]C_{eq}[/tex] is given by,

[tex]C_{eq}[/tex] = ∑C ... (2)

The charge between the plates of the capacitor, q = CV ... (3)

First, we calculate the equivalent capacitance with the capacitors in series. We are given that capacitance on each capacitor is given by, [tex]C_{1}[/tex] = [tex]C_{2}[/tex] = [tex]C_{3}[/tex] = C

Thus, the equivalent capacitance [tex]C_{eq}[/tex] of these capacitors in series is given by:

[tex]\frac{1}{C_{eq} }[/tex] = [tex]\frac{3}{C}C_{eq}[/tex] = [tex]\frac{C}{3}[/tex]

Therefore, the equivalent capacitance [tex]\frac{1}{C_{eq} }[/tex] is [tex]\frac{C}{3}[/tex].

Second, we calculate the equivalent capacitance with the capacitors in parallel. We are given that capacitance on each capacitor is given by, [tex]C_{1}[/tex] = [tex]C_{2}[/tex] = [tex]C_{3}[/tex] = C

Thus, the equivalent capacitance [tex]\frac{1}{C_{eq} }[/tex] of these capacitors in series is given by:

[tex]\frac{1}{C_{eq} }[/tex] = C + C + C = 3C

Therefore, the equivalent capacitance [tex]\frac{1}{C_{eq} }[/tex] is 3C.

Third, we calculate the equivalent capacitance that has more charge. If capacitors are in series, then we get the charge using equivalent capacitance in series using equation (iii) as follows:

q = [tex]\frac{1}{3}CV[/tex]

If capacitors are in parallel, then we get the charge using equivalent capacitance in series using equation (iii) as follows:

q = 3CV

Therefore, we cаn sаy thаt the pаrаllel will end up hаving more chаrge on the equivаlent cаpаcitаnce.

For more information about equivаlent cаpаcitаnce refers to the link: https://brainly.com/question/28445252

#SPJ11

when white light strikes this object, the light is completely absorbed, with none of it transmitted or reflected. which type of object could this be?

Answers

The white light strikes a black object, the light is completely absorbed and none of it is transmitted or reflected.


Light is a type of electromagnetic radiation that travels in straight lines. It is comprised of energy packets that can move through a vacuum. As light passes through a substance or hits an object, it can be affected in various ways.

The interaction between light and an object can be quantified using specific principles of optics.

Light reflection is a phenomenon in which light hits an object and bounces back from its surface.

When light hits a smooth surface like a mirror or water, it bounces back in a regular and predictable way. This type of reflection is called specular reflection.

When light strikes a rough surface, it is reflected in many different directions. This type of reflection is called diffuse reflection.

Therefore, when white light strikes an object and is completely absorbed, with none of it transmitted or reflected, the object can be a black object.

Learn more about White light here:

https://brainly.com/question/166544

#SPJ11

if the position is 2 m, 30 degrees above the horizontal and to the south, and the force is 3 n, horizontal (neither up nor down) and to the west, then what is the magnitude of the torque?

Answers

If the position is 2 m, 30 degrees above the horizontal and to the south, and the force is 3 n, horizontal (neither up nor down) and to the west, then The magnitude of the torque in this scenario is 6 Nm.

The magnitude of the torque in this scenario is determined by calculating the cross product of the position vector and the force vector.

The position vector is given by r = 2m (30° south of the horizontal) and the force vector is given by F = 3N (west).

To calculate the cross product of these two vectors, we can use the formula:

Torque = r x F = |r||F| sin&theta,

where &theta is the angle between the vectors.

In this scenario, the angle between the position vector and the force vector is 90°.

Therefore, the magnitude of the torque can be calculated as follows:

Torque = |r||F|sin90° = (2m)(3N)(1) = 6 Nm.

for such more question on magnitude

https://brainly.com/question/24256733

#SPJ11

a free electron and free proton are released from rest in the same electric field.what is true for the electrical forces experienced by these two particle?

Answers

Answer:

Both particles are subjected to the same electrical force,

The acceleration of the electron will be much greater than that  of the proton:       F = m a and      Mproton / Melectron = 1840

The electron and proton will be accelerated in opposite directions.

what's the field strength on the loop axis at 10.0 cm from the loop center? express your answer in microtesla.

Answers

The field strength on the loop axis at 10.0 cm from the loop center is 0.01 microtesla.

The field strength on the loop axis at 10.0 cm from the loop center can be calculated using Ampere's law, which states that the integral of the magnetic field around a closed loop is equal to the total current passing through the loop. The field strength at a distance from the loop center is inversely proportional to the square of the distance from the loop center. Thus, the field strength on the loop axis at 10.0 cm from the loop center is inversely proportional to 10.0 cm^2 or 100 cm^2, which is equal to 0.01 microtesla.
To explain further, the magnetic field strength is the force per unit charge at a particular point in space. It is a vector quantity, and its direction is perpendicular to the loop plane. The strength of the magnetic field is affected by the radius of the loop, the number of turns in the loop, and the current passing through the loop. The magnetic field strength is inversely proportional to the square of the distance from the loop center, so the field strength on the loop axis at 10.0 cm from the loop center is 0.01 microtesla.

For more such questions on Magnetic field strength.

https://brainly.com/question/11462606#

#SPJ11

Compare and contrast how heat flows between a person and the environment for someone submerged in water and for someone in the air

Answers

Heat transfer between a person and the environment occurs through the processes of convection, conduction, and radiation. The rate of heat transfer depends on factors such as the temperature difference between the person.

What is a conduction ?

Conduction is a process of heat transfer that occurs through a material or between two materials that are in direct contact with each other. In this process, heat flows from a region of higher temperature to a region of lower temperature through molecular collisions. The heat energy is transferred through the material or the contact surface by means of the vibration and movement of the molecules.

Conduction is responsible for heat transfer in solids, such as metals, ceramics, and polymers, and it can also occur between different solids in contact with each other. The rate of conduction depends on several factors, including the thermal conductivity of the material, the temperature difference between the two regions, the thickness of the material, and the surface area of contact.

To know more about Conduction visit :

https://brainly.com/question/12136944

#SPJ1

the attractive forces that exist between gas particles cause the measured pressure of a gas to be lower than that predicted by the ideal gas law true or false

Answers

The attractive forces that exist between gas particles cause the measured pressure of a gas to be lower than that predicted by the ideal gas law. is True because gas particles are in constant motion.

The attractive forces between gas particles are responsible for the deviation of real gases from ideal behavior, causing the pressure to be lower than expected. This is because the ideal gas law assumes that the gas particles are in constant motion and have no intermolecular forces acting upon them.

However, in real gases, there are attractive forces that exist between gas particles, which causes the gas molecules to have less kinetic energy and thus move more slowly. This slower movement leads to a lower pressure than would be predicted by the ideal gas law.

Learn more about ideal gas law at:

https://brainly.com/question/30458409

#SPJ11

Other Questions
what are the three traits of college-level writing? Why did the Soviet army practice a scorched Earth policy while the German armywas invading the Soviet Union?To ensure that the German army could not use any resources that were leftbehindTo intimidate Soviet citizens from helping the Germans as they entered theSoviet UnionTo scare the German army into retreating out of the Soviet UnionTo decrease the likelihood that Germans would attack Soviet civilians the graphic method of note-taking that is especially good at capturing the relationships among ideas is known as the: jamal wants to save 54000 for a odwn payment on a home. how much will he need to invest in an account with 8.2 apr compounding dialy in order to reach his goal in 5y ears which of the following least reflects the purpose of a business rule in the purchase process?multiple choicerequire segregation of order, receiving, and payment dutiesensure suppliers are satisfiedensure an audit trailensure suppliers are paid on time 10 ftGFind the area.8 ftRemember: A = rA = [?] ftRound to the nearesthundredth.Use 3.14 for T. which kind of structure for interest groups have some studies shown may be more engaging for members and more influential on policy at multiple levels? According to the passage, what makes the doer of good glad? what waste product is produced in the krebs cycle? Please helpppp its urgent!!! Assume you have a balance of $3000 on a credit card with an APR of 24 %, or 2 % per month. You start making monthly payments of $200, but at the same time you charge an additional $100 per month to the credit card. Assume that interest for a given month is based on the balance for the previous month. How long does it take to pay off the credit card debt? Round your numbers to the nearest cent? help me plsssWhich function is graphed here?Of(x)=x-2Of(x)=x+2Of(x)=-3x-2Of(x) = -x+2 A circle is shown. Points X and Z are on one side of the circle, and point Y is on the other. Lines are drawn from point Y to point X and from point Y to point Z to form angle X Y Z. Arc X Z is 86 degrees.What is the measure of Y?436886172 What is a chromosome? the earth's lowest barometric pressures are associated with hurricanes (typhoons). frontal systems (cold and warm fronts). sea level. cold and dry climates. very cold temperatures. PLEASE HELP FAST!!!!!!!!!!!!!!!!What was Vietnams economic struggle after the war ended?A.)creating private businesses so residents could earn wages to stimulate the economyB.)reverting factories back to producing and selling military goodsC.)recovering destroyed farms and cities under a new communist economyD.)ending a period of isolation to open communication with other nations for trade HIJ is similar to STR. What is the perimeter of STR? 5 cards are drawn randomly from a regular deck of cards. how many ways can you draw 5 cards and get 4 hearts and 1 spade? If a student had 28 moles of solid carbon would that be enough to produce 15 moles of C2H6 Does a work of art need to be permanent to be considered "real" art?Does it need to be in a form that can be collected and displayed by galleries and museums?How is our experience of an ephemeral piece different than a permanent work? if a maxheap has 100 elements in it, implemented using an array with an empty element at index 0, what is the index of the parent of the element whose index is 47?